1 >>A 20-year-old woman has mild pulmonic stenosis (transvalvular gradient is 20 mmHg). Which of the following statements regarding this situation is true? ?
- (A) Heart size on chest x-ray is likely to be enlarged
- (B) Electrocardiogram is likely to be abnormal
- (C) There is loss of the jugular a wave
- (D) Compared to other valvular defects, the risk of endocarditis is relatively low
2 >>Which of the following findings would be expected in a person with coarctation of the aorta? ?
- (A) Both a systolic murmur and a diastolic murmur would be heard best along the left sternal border
- (B) A higher blood pressure in the left arm than in the right
- (C) Inability to augment cardiac output with exercise
- (D) Often associated with a bicuspid aortic valve
3 >>A 15-year-old boy residing with his parents on a military base presents with a fever of 38.�C (101.5�F) and complains of lower back, knee, and wrist pain. The arthritis is not localized to any one joint. He gives a history of a severe sore throat several weeks earlier. Physical examination of the skin reveals pea-sized swellings over the elbows and wrists. He also has two serpiginous, erythematous pink areas on the anterior trunk, each about 5 cm in diameter. Laboratory investigation includes negative blood cultures, negative throat culture, normal CBC, and an erythrocyte sedimentation rate (ESR) of 100. An antistreptolysin-O (ASO) titer is elevated. At this point, appropriate therapy would consist of ?
- (A) supportive care alone
- (B) parenteral penicillin
- (C) parenteral penicillin and glucocorticoids
- (D) parenteral penicillin and aspirin
4 >>Each patient below is alert and oriented and has a blood pressure of 110/60. In which patient would adenosine constitute appropriate initial therapy? ?
- (A) A 65-year-old man with no ischemic heart disease and wide complex tachycardia
- (B) A 65-year-old woman with known ischemic disease and narrow complex tachycardia
- (C) A 25-year-old woman with known preexcitation syndrome and narrow complex tachycardia
- (D) A 28-year-old man with known preexcitation syndrome and wide complex tachycardia
5 >>A 72-year-old man with a long history of tobacco use presents to the emergency room with shortness of breath. He is mildly cyanotic and hypotensive. On exam he has poor air movement bilaterally and has evidence of an elevated jugular venous pressure. A Swan-Ganz is placed revealing right atrial pressures of 15 mmHg (normal range 0- 5), a pulmonary artery pressure of 70/28 (normal range 12- 28/3- 13), and a pulmonary capillary wedge pressure of 10 (normal range 3- 11). Which of the following is most consistent with this clinical picture? ?
- (A) Inferior myocardial infarction
- (B) Mitral stenosis
- (C) Cor pulmonale
- (D) Mitral regurgitation
6 >>A 23-year-old man presents to the emergency room with palpitations; his heart rate is 180 and he has a normal systolic blood pressure. An attempt to slow his heart rate is made using verapamil. Unfortunately, the patient’s condition rapidly deteriorates. He is now pulseless, and a rhythm strip reveals ventricular fibrillation. Which of the following is most consistent with the patient’s initial cardiac rhythm at presentation? ?
- (A) Atrial flutter
- (B) Sinus tachycardia
- (C) Atrial fibrillation with aberrant conduction
- (D) Atrial fibrillation
7 >>Which of the following statements best describes long-acting nitrate preparations? ?
- (A) Tolerance often develops
- (B) Their effect can be blocked by high doses of beta2 selective inhibitors
- (C) Transdermal patches are more likely to be associated with headaches than are sublingual nitrates
- (D) Oral preparations are more effective than sublingual ones
8 >>A 70-year-old retired banker with no past medical history presents to the emergency department 4 h after the onset of severe substernal crushing chest pain with radiation to the left arm and neck. Electrocardiography reveals significant ST-segment elevation in leads I, L, V , and 5 V . The patient has no clear-cut medical contraindications 6 to anticoagulation. Which of the following would be the optimal management strategy at this time? ?
- (A) Intravenous tissue plasminogen activator alone
- (B) Intravenous tissue plasminogen activator and aspirin
- (C) Intravenous tissue plasminogen activator and heparin
- (D) Intravenous tissue plasminogen activator, heparin, and aspirin
9 >>A 14-year-old boy is brought into the emergency room pulseless and undergoing cardiopulmonary resuscitation. He collapsed while playing basketball. There is no history of trauma, and no prior history of cardiovascular disease. Which of the following conditions would most likely be identified at the time of autopsy? ?
- (A) Normal findings
- (B) Anomalous left coronary artery
- (C) Ruptured aorta
- (D) Hypertrophic cardiomyopathy
10 >>A previously healthy 58-year-old man is admitted to the hospital because of an acute inferior myocardial infarction. Within several hours, he becomes oliguric and hypotensive (blood pressure is 90/60 mmHg). Insertion of a pulmonary artery (Swan-Ganz) catheter reveals the following pressures: pulmonary capillary wedge, 4 mmHg; pulmonary artery, 22/4 mmHg; and mean right atrial, 11 mmHg. This man would best be treated with ?
- (A) fluids
- (B) digoxin
- (C) norepinephrine
- (D) dopamine
11 >>A 63-year-old black woman with a long history of hypertension and diabetes is brought to the emergency department by relatives because she has become incoherent over the past 24 h. Physical examination reveals a disoriented woman whose blood pressure is 230/160, respiratory rate is 25, and pulse is 110. The patient is afebrile. The chest reveals bibasilar rales. Cardiac examination is remarkable only for the presence of an S . There is no 4 organomegaly or focal neurologic findings. The patient is oriented to person only. The family revealed that the patient has not been taking her antihypertensive medicines in the past several weeks. The patient is placed on a cardiac monitor, and both intravenous and intraarterial lines are placed. An emergent CT scan reveals no evidence of hemorrhage or mass lesion. The most appropriate next step in management would be to ?
- (A) observe the patient in a quiet room for 1 h before administering therapy
- (B) wait for laboratory values to return before deciding on specific therapy
- (C) administer sodium nitroprusside
- (D) administer diazoxide
12 >>Which of the following statements regarding physiologic maneuvers used to distinguish one cardiac condition from another is true? ?
- (A) The Valsalva maneuver results in increased length and intensity for most systolic murmurs, except those caused by mitral valve prolapse and hypertrophic cardiomyopathy
- (B) In the case of mitral valve prolapse, squatting results in increased intensity of the systolic murmur
- (C) Handgrip exercise decreases the intensity of the murmurs of mitral stenosis and mitral regurgitation
- (D) Murmurs of tricuspid regurgitation and tricuspid stenosis increase during inspiration
13 >>Which of the following patients should undergo operative excision of an abdominal aortic aneurysm and replacement with a vascular graft? ?
- (A) A 58-year-old man with an 8-cm abdominal aneurysm who sustained a myocardial infarction 3 months ago
- (B) A 65-year-old man with a 7-cm aneurysm who sustained a myocardial infarction 1 year ago
- (C) A 65-year-old woman with a 4-cm aneurysm and no prior history of heart or lung disease
- (D) A 58-year-old man with a 7-cm aneurysm and FEV of 0.8 L
14 >>A 68-year-old man who has had a recent syncopal episode is hospitalized with congestive heart failure. His blood pressure is 160/80 mmHg, his pulse rate is 80 beats per minute, and there is a grade III/VI harsh systolic murmur. An echocardiogram shows a disproportionately thickened ventricular septum and systolic anterior motion of the mitral valve. Which of the following findings would most likely be present in this man? ?
- (A) Radiation of the murmur to the carotid arteries
- (B) Decrease of the murmur with handgrip
- (C) Delayed carotid upstroke
- (D) Signs of mitral stenosis
15 >>A 73-year-old patient develops acute renal failure secondary to bladder obstruction. His serum creatine is 327 ?mol/L (3.7 mg/dL) with a potassium of 6.5 meq/L. The patient had an electrocardiogram, which revealed peaked T waves as the only abnormality. If the hyperkalemia were not corrected, what would be the expected next electrocardiographic abnormality? ?
- (A) T-wave inversion
- (B) PRprolongation and P-wave flattening
- (C) Prolongation of the QT interval
- (D) Widening of the QRS interval
16 >>For the last 6 h, a 33-year-old man has had sharp, pleuritic substernal chest pain that is relieved when he sits upright. His electrocardiogram shows diffuse ST-segment elevation. Which of the following observations supports a diagnosis of acute pericarditis? ?
- (A) ST-segment depression
- (B) PR-segment depression
- (C) Diffuse T-wave inversion with ST-segment elevation
- (D) A normal serum creatine phosphokinase concentration
17 >>Which of the following electrocardiographic findings represents a manifestation of digitalis toxicity? ?
- (A) ST-segment depression
- (B) T-wave inversion
- (C) Atrial flutter
- (D) Atrial tachycardia with variable block
18 >>What is the most common cause of pulseless electrical activity (PEA)? ?
- (A) Acidosis
- (B) Hypovolemia
- (C) Tension pneumothorax
- (D) Pericardial tamponade
19 >>Which of the following congenital cardiac disorders will lead to a left-to-right shunt, generally with cyanosis? ?
- (A) Anomalous origin of the left coronary artery from the pulmonary trunk
- (B) Patent ductus arteriosus without pulmonary hypertension
- (C) Total anomalous pulmonary venous connection
- (D) Ventricular septal defect
20 >>Absolute contraindications to the use of thrombolytic agents in the setting of an acute anterior myocardial infarction include which of the following? ?
- (A) Left carotid artery occlusion with hemiparesis 1 month ago
- (B) Diabetic retinopathy
- (C) Patient on warfarin for atrial fibrillation, with an INRof 1.8
- (D) Patient age > 70
Leave a Reply